Med surg test 2

Ace your homework & exams now with Quizwiz!

D

. The nurse is caring for a patient with hyperparathyroidism. What level of activity would the nurse expect to promote? A) Complete bed rest B) Bed rest with bathroom privileges C) Out of bed (OOB) to the chair twice a day D) Ambulation and activity as tolerated

type 1 anaphylactic reaction

Asthma , allergic rhinitis, insect stings, systemic anaphalaxis

B

A nurse is caring for a patient with type 1 diabetes who is being discharged home tomorrow. What is the best way to assess the patient's ability to prepare and self- administer insulin? A) Ask the patient to describe the process in detail. B) Observe the patient drawing up and administering the insulin. C) Provide a health education session reviewing the main points of insulin delivery. D) Review the patient's first hemoglobin A1C result after discharge

thigh

A nurse is caring for a teenage girl who has had an anaphylactic reaction after a bee sting. The nurse is providing patient teaching prior to the patients discharge. In the event of an anaphylactic reaction, the nurse informs the patient that she should self-administer epinephrine in what site?

D

A nurse is providing health education regarding self-care to a patient with an immunodeficiency. What teaching point should the nurse emphasize? A) The importance of aggressive treatment of acne B) The importance of avoiding alcohol-based cleansers C) The need to keep fingernails and toenails closely trimmed D) The need for thorough oral hygiene

D

A patient has received a diagnosis of type 2 diabetes. The diabetes nurse has made contact with the patient and will implement a program of health education. What is the nurse's priority action? A) Ensure that the patient understands the basic pathophysiology of diabetes. B) Identify the patient's body mass index. C) Teach the patient ìsurvival skillsî for diabetes. D) Assess the patient's readiness to learn.

C

A patient has returned to the floor after having a thyroidectomy for thyroid cancer. The nurse knows that sometimes during thyroid surgery the parathyroid glands can be injured or removed. What laboratory finding may be an early indication of parathyroid gland injury or removal? A) Hyponatremia B) Hypophosphatemia C) Hypocalcemia D) Hypokalemia

A

A patient with Cushing syndrome as a result of a pituitary tumor has been admitted for a transsphenoidal hypophysectomy. What would be most important for the nurse to monitor before, during, and after surgery? A) Blood glucose B) Assessment of urine for blood C) Weight D) Oral temperature

A, C, D

A patient with Cushing syndrome has been hospitalized after a fall. The dietician consulted works with the patient to improve the patient's nutritional intake. What foods should a patient with Cushing syndrome eat to optimize health? Select all that apply. A) Foods high in vitamin D B) Foods high in calories C) Foods high in protein D) Foods high in calcium E) Foods high in sodium

B

A patient with a longstanding diagnosis of generalized anxiety disorder presents to the emergency room. The triage nurse notes upon assessment that the patient is hyperventilating. The triage nurse is aware that hyperventilation is the most common cause of which acidñbase imbalance? A) Respiratory acidosis B) Respiratory alkalosis C) Increased PaCO2 D) CNS disturbances

A

The nurse is caring for a patient with an immunodeficiency who has experienced sudden malaise. The nurse's colleague states, "I'm pretty sure that it's not an infection, because the most recent blood work looks fine." What principle should guide the nurse's response to the colleague? A) Immunodeficient patients will usually exhibit subtle and atypical signs of infection. B) Infections in immunodeficient patients have a slower onset but a more severe course. C) Laboratory blood work is often inaccurate in immunodeficient patients. D) Immunodeficient patients do not develop symptoms of infection.

B

The nurse is discussing macrovascular complications of diabetes with a patient. The nurse would address what topic during this dialogue? A) The need for frequent eye examinations for patients with diabetes B) The fact that patients with diabetes have an elevated risk of myocardial infarction C) The relationship between kidney function and blood glucose levels D) The need to monitor urine for the presence of albumin

A, D

The nurse is performing a shift assessment of a patient with aldosteronism. What assessments should the nurse include? Select all that apply. A) Urine output B) Signs or symptoms of venous thromboembolism C) Peripheral pulses D) Blood pressure E) Skin integrity

Risk for Disturbed Body Image Related to Skin Lesions

The nurse is planning the care of a patient who has a diagnosis of atopic dermatitis, which commonly affects both of her hands and forearms. What risk nursing diagnosis should the nurse include in the patients care plan?

B

The nurse is planning the care of a patient with hyperthyroidism. What should the nurse specify in the patient's meal plan? A) A clear liquid diet, high in nutrients B) Small, frequent meals, high in protein and calories C) Three large, bland meals a day D) A diet high in fiber and plant-sourced fat

D

The nurse is preparing to administer IVIG to a patient who has an immunodeficiency. What nursing guideline should the nurse apply? A) Do not exceed an infusion rate of 300 mL/hr. B) Slow the infusion rate if the patient exhibits signs of a transfusion reaction. C) Weigh the patient immediately after the infusion is complete. D) Administer pretreatment medications as ordered 30 minutes prior to infusion.

Anaphylactic reaction after a bee sting

The nurse is providing care for a patient who has experienced a type I hypersensitivity reaction. What condition is an example of such a reaction?

type 2 cytotoxic reaction

Transfusion reaction, thrombocytopenia, pernicious anemia

A

Two patients on your unit have recently returned to the postsurgical unit after knee arthroplasty. One patient is reporting pain of 8 to 9 on a 0-to-10 pain scale, whereas the other patient is reporting a pain level of 3 to 4 on the same pain scale. What is the nurse's most plausible rationale for understanding the patients' different perceptions of pain? A) Endorphin levels may vary between patients, affecting the perception of pain. B) One of the patients is exaggerating his or her sense of pain. C) The patients are likely experiencing a variance in vasoconstriction. D) One of the patients may be experiencing opioid tolerance

B, C, E

The nurse is caring for a patient at risk for an addisonian crisis. For what associated signs and symptoms should the nurse monitor the patient? Select all that apply. A) Epistaxis B) Pallor C) Rapid respiratory rate D) Bounding pulse E) Hypotension

A

The nurse is caring for a patient diagnosed with hypothyroidism secondary to Hashimoto's thyroiditis. When assessing this patient, what sign or symptom would the nurse expect? A) Fatigue B) Bulging eyes C) Palpitations D) Flushed skin

B

Your patient is receiving postoperative morphine through a patient-controlled analgesic (PCA) pump and the patient's orders specify an initial bolus dose. What is your priority assessment? A) Assessment for decreased level of consciousness (LOC) B) Assessment for respiratory depression C) Assessment for fluid overload D) Assessment for paradoxical increase in pain

type 4 delayed/cellular reaction

occurs 1-3 days after exposure to antigen Contact dermatitis, graft vs host dx; sarcoidosis (inflammatory dx-growth of granulomas ---inflammatory cells; affecting multiple organs but primarily lungs and lymph nodes)

A

A 15-year-old child is brought to the emergency department with symptoms of hyperglycemia and is subsequently diagnosed with diabetes. Based on the fact that the child's pancreatic beta cells are being destroyed, the patient would be diagnosed with what type of diabetes? A) Type 1 diabetes B) Type 2 diabetes C) Nonñinsulin-dependent diabetes D) Prediabetes

B

A 20-year-old patient with an immunodeficiency is admitted to the unit with an acute episode of upper airway edema. This is the fifth time in the past 3 months that the patient has had such as episode. As the nurse caring for this patient, you know that the patient may have a deficiency of what? A) Interferons B) C1esterase inhibitor C) IgG D) IgA

C

A 28-year-old pregnant woman is spilling sugar in her urine. The physician orders a glucose tolerance test, which reveals gestational diabetes. The patient is shocked by the diagnosis, stating that she is conscientious about her health, and asks the nurse what causes gestational diabetes. The nurse should explain that gestational diabetes is a result of what etiologic factor? A) Increased caloric intake during the first trimester B) Changes in osmolality and fluid balance C) The effects of hormonal changes during pregnancy D) Overconsumption of carbohydrates during the first two trimesters

C

A 30 year-old female patient has been diagnosed with Cushing syndrome. What psychosocial nursing diagnosis should the nurse most likely prioritize when planning the patient's care? A) Decisional conflict related to treatment options B) Spiritual distress related to changes in cognitive function C) Disturbed body image related to changes in phy

A, D, E

A gerontologic nurse is teaching students about the high incidence and prevalence of dehydration in older adults. What factors contribute to this phenomenon? Select all that apply. A) Decreased kidney mass B) Increased conservation of sodium C) Increased total body water D) Decreased renal blood flow E) Decreased excretion of potassium

A

A home health nurse is caring for a patient who has an immunodeficiency. What is the nurse's priority action to help ensure successful outcomes and a favorable prognosis? A) Encourage the patient and family to be active partners in the management of the immunodeficiency. B) Encourage the patient and family to manage the patient's activity level and activities of daily living effectively. C) Make sure that the patient and family understand the importance of monitoring fluid balance. D) Make sure that the patient and family know how to adjust dosages of the medications used in treatment.

B

A medical nurse educator is reviewing a patient's recent episode of metabolic acidosis with members of the nursing staff. What should the educator describe about the role of the kidneys in metabolic acidosis? A) The kidneys retain hydrogen ions and excrete bicarbonate ions to help restore balance. B) The kidneys excrete hydrogen ions and conserve bicarbonate ions to help restore balance. C) The kidneys react rapidly to compensate for imbalances in the body. D) The kidneys regulate the bicarbonate level in the intracellular fluid

D

A medical nurse is aware of the need to screen specific patients for their risk of hyperglycemic hyperosmolar syndrome (HHS). In what patient population does hyperosmolar nonketotic syndrome most often occur? A) Patients who are obese and who have no known history of diabetes B) Patients with type 1 diabetes and poor dietary control C) Adolescents with type 2 diabetes and sporadic use of antihyperglycemics D) Middle-aged or older people with either type 2 diabetes or no known history of diabetes

D

A newly admitted patient with type 1 diabetes asks the nurse what caused her diabetes. When the nurse is explaining to the patient the etiology of type 1 diabetes, what process should the nurse describe? A) ìThe tissues in your body are resistant to the action of insulin, making the glucose levels in your blood increase.î B) ìDamage to your pancreas causes an increase in the amount of glucose that it releases, and there is not enough insulin to control it.î C) ìThe amount of glucose that your body makes overwhelms your pancreas and decreases your production of insulin.î D) ìDestruction of special cells in the pancreas causes a decrease in insulin production. Glucose levels rise because insulin normally breaks it down.î

C

A nurse caring for a patient who has an immunosuppressive disorder knows that continual monitoring of the patient is critical. What is the primary rationale behind the need for continual monitoring? A) So that the patient's functional needs can be met immediately B) So that medications can be given as ordered and signs of adverse reactions noted C) So that early signs of impending infection can be detected and treated D) So that the nurse's documentation can be thorough and accurate

The faster the onset of symptoms, the more severe the reaction.

A nurse has asked the nurse educator if there is any way to predict the severity of a patients anaphylactic reaction. What would be the nurses best response?

A

A nurse is admitting a patient with an immunodeficiency to the medical unit. In planning the care of this patient, the nurse should assess for what common sign of immunodeficiency? A) Chronic diarrhea B) Hyperglycemia C) Rhinorrhea D) Contact dermatitis

A

A nurse works in a walk-in clinic. The nurse recognizes that certain patients are at higher risk for different disorders than other patients. What patient is at a greater risk for the development of hypothyroidism? A) A 75-year-old female patient with osteoporosis B) A 50-year-old male patient who is obese C) A 45-year-old female patient who used oral contraceptives D) A 25-year-old male patient who uses recreational drugs

D

A patient with type 2 diabetes has been managing his blood glucose levels using diet and metformin (Glucophage). Following an ordered increase in the patient's daily dose of metformin, the nurse should prioritize which of the following assessments? A) Monitoring the patient's neutrophil levels B) Assessing the patient for signs of impaired liver function C) Monitoring the patient's level of consciousness and behavior D) Reviewing the patient's creatinine and BUN levels

asthma

An adolescent patients history of skin hyperreactivity and inflammation has been attributed to atopic dermatitis. The nurse should recognize that this patient consequently faces an increased risk of what health problem?

B

An elderly patient comes to the clinic with her daughter. The patient is a diabetic and is concerned about foot care. The nurse goes over foot care with the patient and her daughter as the nurse realizes that foot care is extremely important. Why would the nurse feel that foot care is so important to this patient? A) An elderly patient with foot ulcers experiences severe foot pain due to the diabetic polyneuropathy. B) Avoiding foot ulcers may mean the difference between institutionalization and continued independent living. C) Hypoglycemia is linked with a risk for falls; this risk is elevated in older adults with diabetes. D) Oral antihyperglycemics have the possible adverse effect of decreased circulation to the lower extremities

B

An occupational health nurse is screening a group of workers for diabetes. What statement should the nurse interpret as suggestive of diabetes? A) ìI've always been a fan of sweet foods, but lately I'm turned off by them.î B) ìLately, I drink and drink and can't seem to quench my thirst.î C) ìNo matter how much sleep I get, it seems to take me hours to wake up.î D) ìWhen I went to the washroom the last few days, my urine smelled odd.î

B

An occupational health nurse overhears an employee talking to his manager about a 65- year-old coworker. What phenomenon would the nurse identify when hearing the employee state, ìHe should just retire and make way for some new blood.î? A) Intolerance B) Ageism C) Dependence D) Nonspecific prejudice

anaphylactic (type 1)

An office worker takes a cupcake that contains peanut butter. He begins wheezing, with an inspiratory stridor and air hunger and the occupational health nurse is called to the office. The nurse should recognize that the worker is likely suffering from which type of hypersensitivity?

C

As the population of the United States ages, research has shown that this aging will occur across all racial and ethnic groups. A community health nurse is planning an initiative that will focus on the group in which the aging population is expected to rise the fastest. What group should the nurse identify? A) Asian-Americans B) White non-Hispanics C) Hispanics D) African-Americans

intradermal testing

Corticosteroids and antihistamines (including OTC) must be stopped 48-96 hrs before testing becuae they suppress skin reactivity

A, c, d

Falls, which are a major health problem in the elderly population, occur from multifactorial causes. When implementing a comprehensive plan to reduce the incidence of falls on a geriatric unit, what risk factors should nurses identify? Select all that apply. A) Medication effects B) Overdependence on assistive devices C) Poor lighting D) Sensory impairment E) Ineffective use of coping strategies

type 3 immune complex reaction

Fever, joint pain, adenopathy, rash

A

Following an addisonian crisis, a patient's adrenal function has been gradually regained. The nurse should ensure that the patient knows about the need for supplementary glucocorticoid therapy in which of the following circumstances? A) Episodes of high psychosocial stress B) Periods of dehydration C) Episodes of physical exertion D) Administration of a vaccine

D

The baroreceptors, located in the left atrium and in the carotid and aortic arches, respond to changes in the circulating blood volume and regulate sympathetic and parasympathetic neural activity as well as endocrine activities. Sympathetic stimulation constricts renal arterioles, causing what effect? A) Decrease in the release of aldosterone B) Increase of filtration in the Loop of Henle C) Decrease in the reabsorption of sodium D) Decrease in glomerular filtration

D

The home care nurse is conducting patient teaching with a patient on corticosteroid therapy. To achieve consistency with the body's natural secretion of cortisol, when would the home care nurse instruct the patient to take his or her corticosteroids? A) In the evening between 4 PM and 6 PM B) Prior to going to sleep at night C) At noon every day D) In the morning between 7 AM and 8 AM

A

The most recent blood work of a patient with a longstanding diagnosis of type 1 diabetes has shown the presence of microalbuminuria. What is the nurse's most appropriate action? A) Teach the patient about actions to slow the progression of nephropathy. B) Ensure that the patient receives a comprehensive assessment of liver function. C) Determine whether the patient has been using expired insulin. D) Administer a fluid challenge and have the test repeated

B

The nurse caring for a patient post colon resection is assessing the patient on the second postoperative day. The nasogastric tube (NG) remains patent and continues at low intermittent wall suction. The IV is patent and infusing at 125 mL/hr. The patient reports pain at the incision site rated at a 3 on a 0-to-10 rating scale. During your initial shift assessment, the patient complains of cramps in her legs and a tingling sensation in her feet. Your assessment indicates decreased deep tendon reflexes (DTRs) and you suspect the patient has hypokalemia. What other sign or symptom would you expect this patient to exhibit? A) Diarrhea B) Dilute urine C) Increased muscle tone D) Joint pain

C

The nurse educator is differentiating primary immunodeficiency diseases from secondary immunodeficiencies. What is the defining characteristic of primary immunodeficiency diseases? A) They require IVIG as treatment. B) They are the result of intrauterine infection. C) They have a genetic origin. D) They are communicable.

Contraction of bronchial smooth muscle

The nurse in an allergy clinic is educating a new patient about the pathology of the patients health problem. What response should the nurse describe as a possible consequence of histamine release?

C

The nurse in the medical ICU is caring for a patient who is in respiratory acidosis due to inadequate ventilation. What diagnosis could the patient have that could cause inadequate ventilation? A) Endocarditis B) Multiple myeloma C) Guillain-BarrÈ syndrome D) Overdose of amphetamines

A, D, E

The nurse is applying standard precautions in the care of a patient who has an immunodeficiency. What are key elements of standard precautions? Select all that apply. A) Using appropriate personal protective equipment B) Placing patients in negative-pressure isolation rooms C) Placing patients in positive-pressure isolation rooms D) Using safe injection practices E) Performing hand hygiene

C

The nurse is assessing a patient diagnosed with Graves' disease. What physical characteristics of Graves' disease would the nurse expect to find? A) Hair loss B) Moon face C) Bulging eyes D) Fatigue

D

When planning the care of a patient with a fluid imbalance, the nurse understands that in the human body, water and electrolytes move from the arterial capillary bed to the interstitial fluid. What causes this to occur? A) Active transport of hydrogen ions across the capillary walls B) Pressure of the blood in the renal capillaries C) Action of the dissolved particles contained in a unit of blood D) Hydrostatic pressure resulting from the pumping action of the heart

C

Which of the following patients with type 1 diabetes is most likely to experience adequate glucose control? A) A patient who skips breakfast when his glucose reading is greater than 220 mg/dL B) A patient who never deviates from her prescribed dose of insulin C) A patient who adheres closely to a meal plan and meal schedule D) A patient who eliminates carbohydrates from his daily intake

C

You are called to your patient's room by a family member who voices concern about the patient's status. On assessment, you find the patient tachypnic, lethargic, weak, and exhibiting a diminished cognitive ability. You also find 3+ pitting edema. What electrolyte imbalance is the most plausible cause of this patient's signs and symptoms? A) Hypocalcemia B) Hyponatremia C) Hyperchloremia D) Hypophosphatemia

D

You are caring for a patient who is being treated on the oncology unit with a diagnosis of lung cancer with bone metastases. During your assessment, you note the patient complains of a new onset of weakness with abdominal pain. Further assessment suggests that the patient likely has a fluid volume deficit. You should recognize that this patient may be experiencing what electrolyte imbalance? A) Hypernatremia B) Hypomagnesemia C) Hypophosphatemia D) Hypercalcemia

B

You are caring for a patient with a diagnosis of pancreatitis. The patient was admitted from a homeless shelter and is a vague historian. The patient appears malnourished and on day 3 of the patient's admission total parenteral nutrition (TPN) has been started. Why would you know to start the infusion of TPN slowly? A) Patients receiving TPN are at risk for hypercalcemia if calories are started too rapidly. B) Malnourished patients receiving parenteral nutrition are at risk for hypophosphatemia if calories are started too aggressively. C) Malnourished patients who receive fluids too rapidly are at risk for hypernatremia. D) Patients receiving TPN need a slow initiation of treatment in order to allow digestive enzymes to accumulate

D

You are caring for a patient with a secondary diagnosis of hypermagnesemia. What assessment finding would be most consistent with this diagnosis? A) Hypertension B) Kussmaul respirations C) Increased DTRs D) Shallow respirations

A

You are the nurse caring for a 77-year-old male patient who has been involved in a motor vehicle accident. You and your colleague note that the patient's labs indicate minimally elevated serum creatinine levels, which your colleague dismisses. What can this increase in creatinine indicate in older adults? A) Substantially reduced renal function B) Acute kidney injury C) Decreased cardiac output D) Alterations in ratio of body fluids to muscle mass

A

You are the nurse caring for a patient who is to receive IV daunorubicin, a chemotherapeutic agent. You start the infusion and check the insertion site as per protocol. During your most recent check, you note that the IV has infiltrated so you stop the infusion. What is your main concern with this infiltration? A) Extravasation of the medication B) Discomfort to the patient C) Blanching at the site D) Hypersensitivity reaction to the medication

B, C, D

A gerontologic nurse has been working hard to change the perceptions of the elderly, many of which are negative, by other segments of the population. What negative perceptions of older people have been identified in the literature? Select all that apply. A) As being the cause of social problems B) As not contributing to society C) As draining economic resources D) As competing with children for resources E) As dominating health care research

C

A nurse educator is reviewing peripheral IV insertion with a group of novice nurses. How should these nurses be encouraged to deal with excess hair at the intended site? A) Leave the hair intact. B) Shave the area. C) Clip the hair in the area. D) Remove the hair with a depilatory

C

After a sudden decline in cognition, a 77-year-old man who has been diagnosed with vascular dementia is receiving care in his home. To reduce this man's risk of future infarcts, what action should the nurse most strongly encourage? A) Activity limitation and falls reduction efforts B) Adequate nutrition and fluid intake C) Rigorous control of the patient's blood pressure and serum lipid levels D) Use of mobility aids to promote independence

A

For several years, a community health nurse has been working with a 78-year-old man who requires a wheelchair for mobility. The nurse is aware that the interactions between disabilities and aging are not yet clearly understood. This interaction varies, depending on what variable? A) Socioeconomics B) Ethnicity C) Education D) Pharmacotherapy

B

Mrs. Harris is an 83-year-old woman who has returned to the community following knee replacement surgery. The community health nurse recognizes that Mrs. Harris has prescriptions for nine different medications for the treatment of varied health problems. In addition, she has experienced occasional episodes of dizziness and lightheadedness since her discharge. The nurse should identify which of the following nursing diagnoses? A) Risk for infection related to polypharmacy and hypotension B) Risk for falls related to polypharmacy and impaired balance C) Adult failure to thrive related to chronic disease and circulatory disturbance D) Disturbed thought processes related to adverse drug effects and hypotension

B

The admissions department at a local hospital is registering an elderly man for an outpatient diagnostic test. The admissions nurse asks the man if he has an advanced directive. The man responds that he does not want to complete an advance directive because he does not want anyone controlling his finances. What would be appropriate information for the nurse to share with this patient? A) ìAdvance directives are not legal documents, so you have nothing to worry about.î B) ìAdvance directives are limited only to health care instructions and directives.î C) ìYour finances cannot be managed without an advance directive.î D) ìAdvance directives are implemented when you become incapacitated, and then you will use a living will to allow the state to manage your money.î

B, D, E

The case manager is working with an 84-year-old patient newly admitted to a rehabilitation facility. When developing a care plan for this older adult, which factors should the nurse identify as positive attributes that benefit coping in this age group? Select all that apply. A) Decreased risk taking B) Effective adaptation skills C) Avoiding participation in untested roles D) Increased life experience E) Resiliency during change

B

The wife of a patient you are caring for asks to speak with you. She tells you that she is concerned because her husband is requiring increasingly high doses of analgesia. She states, ìHe was in pain long before he got cancer because he broke his back about 20 years ago. For that problem, though, his pain medicine wasn't just raised and raised.î What would be the nurses' best response? A) ìI didn't know that. I will speak to the doctor about your husband's pain control.î B) ìMuch cancer pain is caused by tumor involvement and needs to be treated in a way that brings the patient relief.î C) ìCancer is a chronic kind of pain so the more it hurts the patient, the more medicine we give the patient until it no longer hurts.î D) ìDoes the increasing medication dosage concern you?

D

You are an emergency-room nurse caring for a trauma patient. Your patient has the following arterial blood gas results: pH 7.26, PaCO2 28, HCO3 11 mEq/L. How would you interpret these results? A) Respiratory acidosis with no compensation B) Metabolic alkalosis with a compensatory alkalosis C) Metabolic acidosis with no compensation D) Metabolic acidosis with a compensatory respiratory alkalosis

C

Your patient has just returned from the postanesthetic care unit (PACU) following left tibia open reduction internal fixation (ORIF). The patient is complaining of pain, and you are preparing to administer the patient's first scheduled dose of hydromorphone (Dilaudid). Prior to administering the drug, you would prioritize which of the following assessments? A) The patient's electrolyte levels B) The patient's blood pressure C) The patient's allergy status D) The patient's hydration status

A pregnant woman at 30 weeks gestation

A nurse knows of several patients who have achieved adequate control of their allergy symptoms using over-the-counter antihistamines. Antihistamines would be contraindicated in the care of which patient?

C

. A nurse caring for a patient with diabetes insipidus is reviewing laboratory results. What is an expected urinalysis finding? A) Glucose in the urine B) Albumin in the urine C) Highly dilute urine D) Leukocytes in the urine

C

. A patient has been admitted to the post-surgical unit following a thyroidectomy. To promote comfort and safety, how should the nurse best position the patient? A) Side-lying (lateral) with one pillow under the head B) Head of the bed elevated 30 degrees and no pillows placed under the head C) Semi-Fowler's with the head supported on two pillows D) Supine, with a small roll supporting the neck

C, E

. A patient has been assessed for aldosteronism and has recently begun treatment. What are priority areas for assessment that the nurse should frequently address? Select all that apply. A) Pupillary response B) Creatinine and BUN levels C) Potassium level D) Peripheral pulses E) BP

C

. A patient is prescribed corticosteroid therapy. What would be priority information for the nurse to give the patient who is prescribed long-term corticosteroid therapy? A) The patient's diet should be low protein with ample fat. B) The patient may experience short-term changes in cognition. C) The patient is at an increased risk for developing infection. D) The patient is at a decreased risk for development of thrombophlebitis and thromboembolism.

B

. A patient who has been taking corticosteroids for several months has been experiencing muscle wasting. The patient has asked the nurse for suggestions to address this adverse effect. What should the nurse recommend? A) Activity limitation to conserve energy B) Consumption of a high-protein diet C) Use of OTC vitamin D and calcium supplements D) Passive range-of-motion exercises

D

. A patient with a recent diagnosis of hypothyroidism is being treated for an unrelated injury. When administering medications to the patient, the nurse should know that the patient's diminished thyroid function may have what effect? A) Anaphylaxis B) Nausea and vomiting C) Increased risk of drug interactions D) Prolonged duration of effect

C

. The nurse caring for a patient with Cushing syndrome is describing the dexamethasone suppression test scheduled for tomorrow. What does the nurse explain that this test will involve? A) Administration of dexamethasone orally, followed by a plasma cortisol level every hour for 3 hours B) Administration of dexamethasone IV, followed by an x-ray of the adrenal glands C) Administration of dexamethasone orally at 11 PM, and a plasma cortisol level at 8 AM the next morning D) Administration of dexamethasone intravenously, followed by a plasma cortisol level 3 hours after the drug is administered

B

. The physician has ordered a fluid deprivation test for a patient suspected of having diabetes insipidus. During the test, the nurse should prioritize what assessments? A) Temperature and oxygen saturation B) Heart rate and BP C) Breath sounds and bowel sounds D) Color, warmth, movement, and sensation of extremities

B

. What should the nurse teach a patient on corticosteroid therapy in order to reduce the patient's risk of adrenal insufficiency? A) Take the medication late in the day to mimic the body's natural rhythms. B) Always have enough medication on hand to avoid running out. C) Skip up to 2 doses in cases of illness involving nausea. D) Take up to 1 extra dose per day during times of stress

C

A diabetes educator is teaching a patient about type 2 diabetes. The educator recognizes that the patient understands the primary treatment for type 2 diabetes when the patient states what? A) ìI read that a pancreas transplant will provide a cure for my diabetes.î B) ìI will take my oral antidiabetic agents when my morning blood sugar is high.î C) ìI will make sure to follow the weight loss plan designed by the dietitian.î D) ìI will make sure I call the diabetes educator when I have questions about my insulin.î

D

A diabetes nurse educator is presenting the American Diabetes Association (ADA) recommendations for levels of caloric intake. What do the ADA's recommendations include? A) 10% of calories from carbohydrates, 50% from fat, and the remaining 40% from protein B) 10% to 20% of calories from carbohydrates, 20% to 30% from fat, and the remaining 50% to 60% from protein C) 20% to 30% of calories from carbohydrates, 50% to 60% from fat, and the remaining 10% to 20% from protein D) 50% to 60% of calories from carbohydrates, 20% to 30% from fat, and the remaining 10% to 20% from protein

A

A diabetes nurse educator is teaching a group of patients with type 1 diabetes about ìsick day rules.î What guideline applies to periods of illness in a diabetic patient? A) Do not eliminate insulin when nauseated and vomiting. B) Report elevated glucose levels greater than 150 mg/dL. C) Eat three substantial meals a day, if possible. D) Reduce food intake and insulin doses in times of illness

B

A diabetes nurse is assessing a patient's knowledge of self-care skills. What would be the most appropriate way for the educator to assess the patient's knowledge of nutritional therapy in diabetes? A) Ask the patient to describe an optimally healthy meal. B) Ask the patient to keep a food diary and review it with the nurse. C) Ask the patient's family what he typically eats. D) Ask the patient to describe a typical day's food intake

A

A diabetic educator is discussing ìsick day rulesî with a newly diagnosed type 1 diabetic. The educator is aware that the patient will require further teaching when the patient states what? A) ìI will not take my insulin on the days when I am sick, but I will certainly check my blood sugar every 2 hours.î B) ìIf I cannot eat a meal, I will eat a soft food such as soup, gelatin, or pudding six to eight times a day.î C) ìI will call the doctor if I am not able to keep liquids in my body due to vomiting or diarrhea.î D) ìI will call the doctor if my blood sugar is over 300 mg/dL or if I have ketones in my urine.î

A

A diabetic nurse is working for the summer at a camp for adolescents with diabetes. When providing information on the prevention and management of hypoglycemia, what action should the nurse promote? A) Always carry a form of fast-acting sugar. B) Perform exercise prior to eating whenever possible. C) Eat a meal or snack every 8 hours. D) Check blood sugar at least every 24 hours.

B

A diabetic patient calls the clinic complaining of having a ìflu bug.î The nurse tells him to take his regular dose of insulin. What else should the nurse tell the patient? A) ìMake sure to stick to your normal diet.î B) ìTry to eat small amounts of carbs, if possible.î C) ìEnsure that you check your blood glucose every hour.î D) ìFor now, check your urine for ketones every 8 hours.î

B

A medical nurse is caring for a patient with type 1 diabetes. The patient's medication administration record includes the administration of regular insulin three times daily. Knowing that the patient's lunch tray will arrive at 11:45, when should the nurse administer the patient's insulin? A) 10:45 B) 11:15 C) 11:45 D) 11:50

cytotoxic type2

A patient is receiving a transfusion of packed red blood cells. Shortly after initiation of the transfusion, the patient begins to exhibit signs and symptoms of a transfusion reaction. The patient is suffering from which type of hypersensitivity?

B

A nurse has created a plan of care for an immunodeficient patient, specifying that care providers take the patient's pulse and respiratory rate for a full minute. What is the rationale for this aspect of care? A) Respirations affect heart rate in immunodeficient patients. B) These patients' blunted inflammatory responses can cause subtle changes in status. C) Hemodynamic instability is one of the main complications of immunodeficiency. D) Immunodeficient patients are prone to ventricular tachycardia and atrial fibrillation.

D

A nurse is assessing a patient who has diabetes for the presence of peripheral neuropathy. The nurse should question the patient about what sign or symptom that would suggest the possible development of peripheral neuropathy? A) Persistently cold feet B) Pain that does not respond to analgesia C) Acute pain, unrelieved by rest D) The presence of a tingling sensation

A

A nurse is caring for a patient newly diagnosed with type 1 diabetes. The nurse is educating the patient about self-administration of insulin in the home setting. The nurse should teach the patient to do which of the following? A) Avoid using the same injection site more than once in 2 to 3 weeks. B) Avoid mixing more than one type of insulin in a syringe. C) Cleanse the injection site thoroughly with alcohol prior to injecting. D) Inject at a 45∫ angle

Modify the environment to reduce the severity of allergic symptoms.

A nurse is caring for a patient who has allergic rhinitis. What intervention would be most likely to help the patient meet the goal of improved breathing pattern?

B

A nurse is caring for a patient who has an immunodeficiency. What assessment finding should prompt the nurse to consider the possibility that the patient is developing an infection? A) Uncharacteristic aggression B) Persistent diarrhea C) Pruritis (itching) D) Constipation

B

A nurse is caring for a patient with a phagocytic cell disorder. The patient states, "My specialist says that I will likely be cured after I get my treatment tomorrow." To what treatment is the patient most likely referring? A) Treatment with granulocyte-macrophage colony-stimulating factor (GM-CSF) B) Hematopoietic stem cell transplantation C) Treatment with granulocyte colony-stimulating factor (G-CSF) D) Brachytherapy

A

A nurse is conducting a class on how to self-manage insulin regimens. A patient asks how long a vial of insulin can be stored at room temperature before it ìgoes bad.î What would be the nurse's best answer? A) ìIf you are going to use up the vial within 1 month it can be kept at room temperature.î B) ìIf a vial of insulin will be used up within 21 days, it may be kept at room temperature.î C) ìIf a vial of insulin will be used up within 2 weeks, it may be kept at room temperature.î D) ìIf a vial of insulin will be used up within 1 week, it may be kept at room temperature.î

D

A nurse is planning the care of a patient who requires immunosuppression to ensure engraftment of depleted bone marrow during a transplantation procedure. What is the most important component of infection control in the care of this patient? A) Administration of IVIG B) Antibiotic administration C) Appropriate use of gloves and goggles D) Thorough and consistent hand hygiene

Emergency equipment should be readily available.

A nurse is preparing a patient for allergy skin testing. Which of the following precautionary steps is most important for the nurse to follow?

A

A nurse is preparing to administer a scheduled dose of IVIG to a patient who has a diagnosis of severe combined immunodeficiency disease (SCID). What medication should the nurse administer prior to initiating the infusion? A) Diphenhydramine B) Ibuprofen C) Hydromorphone D) Fentanyl

B

A nurse is preparing to discharge a patient with an immunodeficiency. When preparing the patient for self-infusion of IVIG in the home setting, what education should the nurse prioritize? A) Sterile technique for establishing a new IV site B) Signs and symptoms of adverse reactions C) Formulas for calculating daily doses D) Technique for adding medications to the IVIG

D

A nurse is teaching basic ìsurvival skillsî to a patient newly diagnosed with type 1 diabetes. What topic should the nurse address? A) Signs and symptoms of diabetic nephropathy B) Management of diabetic ketoacidosis C) Effects of surgery and pregnancy on blood sugar levels D) Recognition of hypoglycemia and hyperglycemia

B, D

A patient has been admitted to the critical care unit with a diagnosis of thyroid storm. What interventions should the nurse include in this patient's immediate care? Select all that apply. A) Administering diuretics to prevent fluid overload B) Administering beta blockers to reduce heart rate C) Administering insulin to reduce blood glucose levels D) Applying interventions to reduce the patient's temperature E) Administering corticosteroids

increased eosinophils

A patient has been admitted to the emergency department with signs of anaphylaxis following a bee sting. The nurse knows that if this is a true allergic reaction the patient will present with what alteration in laboratory values?

B, C, D

A patient has been admitted with a phagocytic cell disorder and the nurse is reviewing the most common health problems that accompany these disorders. The nurse should identify which of the following? Select all that apply. A) Inflammatory bowel disease B) Chronic otitis media C) Cutaneous abscesses D) Pneumonia E) Cognitive deficits

foods, medications, insect stings

A patient has been brought to the emergency department by EMS after being found unresponsive. Rapid assessment reveals anaphylaxis as a potential cause of the patients condition. The care team should attempt to assess for what potential causes of anaphylaxis?

A

A patient has been brought to the emergency department by paramedics after being found unconscious. The patient's Medic Alert bracelet indicates that the patient has type 1 diabetes and the patient's blood glucose is 22 mg/dL (1.2 mmol/L). The nurse should anticipate what intervention? A) IV administration of 50% dextrose in water B) Subcutaneous administration of 10 units of Humalog C) Subcutaneous administration of 12 to 15 units of regular insulin D) IV bolus of 5% dextrose in 0.45% NaCl

A

A patient has been living with type 2 diabetes for several years, and the nurse realizes that the patient is likely to have minimal contact with the health care system. In order to ensure that the patient maintains adequate blood sugar control over the long term, the nurse should recommend which of the following? A) Participation in a support group for persons with diabetes B) Regular consultation of websites that address diabetes management C) Weekly telephone ìcheck-insî with an endocrinologist D) Participation in clinical trials relating to antihyperglycemics

D

A patient has been taking prednisone for several weeks after experiencing a hypersensitivity reaction. To prevent adrenal insufficiency, the nurse should ensure that the patient knows to do which of the following? A) Take the drug concurrent with levothyroxine (Synthroid). B) Take each dose of prednisone with a dose of calcium chloride. C) Gradually replace the prednisone with an OTC alternative. D) Slowly taper down the dose of prednisone, as ordered.

Keep her hands well-moisturized at all times.

A patient has developed severe contact dermatitis with burning, itching, cracking, and peeling of the skin on her hands. What should the nurse teach the patient to do?

B

A patient has just been diagnosed with type 2 diabetes. The physician has prescribed an oral antidiabetic agent that will inhibit the production of glucose by the liver and thereby aid in the control of blood glucose. What type of oral antidiabetic agent did the physician prescribe for this patient? A) A sulfonylurea B) A biguanide C) A thiazolidinedione D) An alpha glucosidase inhibitor

identifying offending agent if possible

A patient has presented with signs and symptoms that are consistent with contact dermatitis. What aspect of care should the nurse prioritize when working with this patient?

A

A patient has questioned the nurse's administration of IV normal saline, asking whether sterile water would be a more appropriate choice than ìsaltwater.î Under what circumstances would the nurse administer electrolyte-free water intravenously? A) Never, because it rapidly enters red blood cells, causing them to rupture. B) When the patient is severely dehydrated resulting in neurologic signs and symptoms C) When the patient is in excess of calcium and/or magnesium ions D) When a patient's fluid volume deficit is due to acute or chronic renal failure

type 1

A patient has sought care, stating that she developed hives overnight. The nurses inspection confirms the presence of urticaria. What type of allergic hypersensitivity reaction has the patient developed?

A

A patient is admitted for the treatment of a primary immunodeficiency and intravenous immunoglobulin (IVIG) is ordered. What should the nurse monitor for as a potential adverse effect of IVIG administration? A) Anaphylaxis B) Hypertension C) Hypothermia D) Joint pain

B, C, D, E

A patient is brought to the emergency department by the paramedics. The patient is a type 2 diabetic and is experiencing HHS. The nurse should identify what components of HHS? Select all that apply. A) Leukocytosis B) Glycosuria C) Dehydration D) Hypernatremia E) Hyperglycemia

montelukast (singular)

A patient is learning about his new diagnosis of asthma with the asthma nurse. What medication has the ability to prevent the onset of acute asthma exacerbations?

A

A patient is undergoing testing for suspected adrenocortical insufficiency. The care team should ensure that the patient has been assessed for the most common cause of adrenocortical insufficiency. What is the most common cause of this health problem? A) Therapeutic use of corticosteroids B) Pheochromocytoma C) Inadequate secretion of ACTH D) Adrenal tumor

C

A patient newly diagnosed with type 2 diabetes is attending a nutrition class. What general guideline would be important to teach the patients at this class? A) Low fat generally indicates low sugar. B) Protein should constitute 30% to 40% of caloric intake. C) Most calories should be derived from carbohydrates. D) Animal fats should be eliminated from the diet.

D

A patient on corticosteroid therapy needs to be taught that a course of corticosteroids of 2 weeks' duration can suppress the adrenal cortex for how long? A) Up to 4 weeks B) Up to 3 months C) Up to 9 months D) Up to 1 year

A

A patient presents at the walk-in clinic complaining of diarrhea and vomiting. The patient has a documented history of adrenal insufficiency. Considering the patient's history and current symptoms, the nurse should anticipate that the patient will be instructed to do which of the following? A) Increase his intake of sodium until the GI symptoms improve. B) Increase his intake of potassium until the GI symptoms improve. C) Increase his intake of glucose until the GI symptoms improve. D) Increase his intake of calcium until the GI symptoms improve.

A

A patient presents to the clinic complaining of symptoms that suggest diabetes. What criteria would support checking blood levels for the diagnosis of diabetes? A) Fasting plasma glucose greater than or equal to 126 mg/dL B) Random plasma glucose greater than 150 mg/dL C) Fasting plasma glucose greater than 116 mg/dL on 2 separate occasions D) Random plasma glucose greater than 126 mg/dL

D

A patient who has received a heart transplant is taking cyclosporine, an immunosuppressant. What should the nurse emphasize during health education about infection prevention? A) Eat a high-calorie, high-protein diet. B) Limit physical activity in order to conserve energy. C) Take prophylactic antibiotics as ordered. D) Perform frequent handwashing.

The patients test should be cancelled until he is off his corticosteroids.

A patient who is scheduled for a skin test informs the nurse that he has been taking corticosteroids to help control his allergy symptoms. What nursing intervention should the nurse implement?

C

A patient with a diagnosis of primary immunodeficiency informs the nurse that he has been experiencing a new onset of a dry cough and occasional shortness of breath. After determining that the patient's vital signs are within reference ranges, what action should the nurse take? A) Administer a nebulized bronchodilator. B) Perform oral suctioning. C) Assess the patient for signs and symptoms of infection. D) Teach the patient deep breathing and coughing exercises.

B

A patient with a diagnosis of syndrome of inappropriate antidiuretic hormone secretion (SIADH) is being cared for on the critical care unit. The priority nursing diagnosis for a patient with this condition is what? A) Risk for peripheral neurovascular dysfunction B) Excess fluid volume C) Hypothermia D) Ineffective airway clearance

B

A patient with a history of type 1 diabetes has just been admitted to the critical care unit (CCU) for diabetic ketoacidosis. The CCU nurse should prioritize what assessment during the patient's initial phase of treatment? A) Monitoring the patient for dysrhythmias B) Maintaining and monitoring the patient's fluid balance C) Assessing the patient's level of consciousness D) Assessing the patient for signs and symptoms of venous thromboembolism

A

A patient with a longstanding diagnosis of type 1 diabetes has a history of poor glycemic control. The nurse recognizes the need to assess the patient for signs and symptoms of peripheral neuropathy. Peripheral neuropathy constitutes a risk for what nursing diagnosis? A) Infection B) Acute pain C) Acute confusion D) Impaired urinary elimination

D

A patient with hypofunction of the adrenal cortex has been admitted to the medical unit. What would the nurse most likely find when assessing this patient? A) Increased body temperature B) Jaundice C) Copious urine output D) Decreased BP

D

A patient with pheochromocytoma has been admitted for an adrenalectomy to be performed the following day. To prevent complications, the nurse should anticipate preoperative administration of which of the following? A) IV antibiotics B) Oral antihypertensives C) Parenteral nutrition D) IV corticosteroids

The patient will remain in the clinic to be monitored for 30 minutes following the injection.

A patient with severe environmental allergies is scheduled for an immunotherapy injection. What should be included in teaching the patient about this treatment?

A

A patient with suspected adrenal insufficiency has been ordered an adrenocorticotropic hormone (ACTH) stimulation test. Administration of ACTH caused a marked increase in cortisol levels. How should the nurse interpret this finding? A) The patient's pituitary function is compromised. B) The patient's adrenal insufficiency is not treatable. C) The patient has insufficient hypothalamic function. D) The patient would benefit from surgery

A

A patient with thyroid cancer has undergone surgery and a significant amount of parathyroid tissue has been removed. The nurse caring for the patient should prioritize what question when addressing potential complications? A) ìDo you feel any muscle twitches or spasms?î B) ìDo you feel flushed or sweaty?î C) ìAre you experiencing any dizziness or lightheadedness?î D) ìAre you having any pain that seems to be radiating from your bones?î

C

A patient with type 1 diabetes has told the nurse that his most recent urine test for ketones was positive. What is the nurse's most plausible conclusion based on this assessment finding? A) The patient should withhold his next scheduled dose of insulin. B) The patient should promptly eat some protein and carbohydrates. C) The patient's insulin levels are inadequate. D) The patient would benefit from a dose of metformin (Glucophage)

C

A patient with type 1 diabetes mellitus is seeing the nurse to review foot care. What would be a priority instruction for the nurse to give the patient? A) Examine feet weekly for redness, blisters, and abrasions. B) Avoid the use of moisturizing lotions. C) Avoid hot-water bottles and heating pads. D) Dry feet vigorously after each bath

B

A patient with type 2 diabetes achieves adequate glycemic control through diet and exercise. Upon being admitted to the hospital for a cholecystectomy, however, the patient has required insulin injections on two occasions. The nurse would identify what likely cause for this short-term change in treatment? A) Alterations in bile metabolism and release have likely caused hyperglycemia. B) Stress has likely caused an increase in the patient's blood sugar levels. C) The patient has likely overestimated her ability to control her diabetes using nonpharmacologic measures. D) The patient's volatile fluid balance surrounding surgery has likely caused unstable blood sugars

D

A patient's most recent laboratory results show a slight decrease in potassium. The physician has opted to forego drug therapy but has suggested increasing the patient's dietary intake of potassium. Which of the following would be a good source of potassium? A) Apples B) Asparagus C) Carrots D) Bananas

C

A patient's primary immunodeficiency disease is characterized by the inability of white blood cells to initiate an inflammatory response to infectious organisms. What is this patient's most likely diagnosis? A) Chronic granulomatous disease B) Wiskott-Aldrich syndrome C) Hyperimmunoglobulinemia E syndrome D) Common variable immunodeficiency

The patients body has mistakenly identified a normal constituent of the body as foreign.

A patients decline in respiratory and renal function has been attributed to Goodpasture syndrome, which is a type II hypersensitivity reaction. What pathologic process underlies the patients health problem?

C

A physician has explained to a patient that he has developed diabetic neuropathy in his right foot. Later that day, the patient asks the nurse what causes diabetic neuropathy. What would be the nurse's best response? A) ìResearch has shown that diabetic neuropathy is caused by fluctuations in blood sugar that have gone on for years.î B) ìThe cause is not known for sure but it is thought to have something to do with ketoacidosis.î C) ìThe cause is not known for sure but it is thought to involve elevated blood glucose levels over a period of years.î D) ìResearch has shown that diabetic neuropathy is caused by a combination of elevated glucose levels and elevated ketone levels.î

D

A school nurse is teaching a group of high school students about risk factors for diabetes. Which of the following actions has the greatest potential to reduce an individual's risk for developing diabetes? A) Have blood glucose levels checked annually. B) Stop using tobacco in any form. C) Undergo eye examinations regularly. D) Lose weight, if obese

C

A student with diabetes tells the school nurse that he is feeling nervous and hungry. The nurse assesses the child and finds he has tachycardia and is diaphoretic with a blood glucose level of 50 mg/dL (2.8 mmol/L). What should the school nurse administer? A) A combination of protein and carbohydrates, such as a small cup of yogurt B) Two teaspoons of sugar dissolved in a cup of apple juice C) Half of a cup of juice, followed by cheese and crackers D) Half a sandwich with a protein-based filling

C

A teenager is diagnosed with cellulitis of the right knee and fails to respond to oral antibiotics. He then develops osteomyelitis of the right knee, prompting a detailed diagnostic workup that reveals a phagocytic disorder. This patient faces an increased risk of what complication? A) Thrombocytopenia B) HIV/AIDS C) Neutropenia D) Hemophilia

D

An older adult patient with type 2 diabetes is brought to the emergency department by his daughter. The patient is found to have a blood glucose level of 623 mg/dL. The patient's daughter reports that the patient recently had a gastrointestinal virus and has been confused for the last 3 hours. The diagnosis of hyperglycemic hyperosmolar syndrome (HHS) is made. What nursing action would be a priority? A) Administration of antihypertensive medications B) Administering sodium bicarbonate intravenously C) Reversing acidosis by administering insulin D) Fluid and electrolyte replacement

D

Diagnostic testing has been ordered to differentiate between normal anion gap acidosis and high anion gap acidosis in an acutely ill patient. What health problem typically precedes normal anion gap acidosis? A) Metastases B) Excessive potassium intake C) Water intoxication D) Excessive administration of chloride

A

Family members of an immunocompromised patient have asked the nurse why antibiotics are not being given to the patient in order to prevent infection. How should the nurse best respond? A) "Using antibiotics to prevent infections can cause the growth of drug-resistant bacteria." B) "If an antibiotic is given to prevent a bacterial infection, the patient is at risk of a viral infection." C) "Antibiotics can never prevent an infection; they can only cure an infection that is fully developed." D) "Antibiotics cannot resolve infections in people who are immunocompromised."

s/s anaphylaxis

Flushing Urticaria Angioedema Hypotention Bronchoconstriction

D

IVIG has been ordered for the treatment of a patient with an immunodeficiency. Which of the following actions should the nurse perform before administering this blood product? A) Ensure that the patient has a patent central line. B) Ensure that the IVIG is appropriately mixed with normal saline. C) Administer furosemide before IVIG to prevent hypervolemia. D) Weigh the patient before administration to verify the correct dose.

Hypoglycemia

Initial treatment for ___ is 15 g concentrated carbohydrate, such as two or three glucose tablets, 1 tube glucose gel, or 0.5 cup juice. After initial treatment, the nurse should follow with a snack including starch and protein, such as cheese and crackers, milk and crackers, or half of a sandwich. It is unnecessary to add sugar to juice, even it if is labeled as unsweetened juice, because the fruit sugar in juice contains enough simple carbohydrate to raise the blood glucose level and additional sugar may result in a sharp rise in blood sugar that will last for several hours

A

Patient teaching regarding infection prevention for the patient with an immunodeficiency includes which of the following guidelines? A) Cook all food thoroughly. B) Refrain from using creams or emollients on skin. C) Maintain contact only with individuals who have recently been vaccinated. D) Take OTC vitamin supplements consistently.

allergic disorder skin tests

Skin tests: note precautions!!!!!!!!!!!!!------------do not perform during bronchosparm; scratch or prick tests are performed before other testing methods in order to minimize the risk of systemic reaction; ER equipment must be ready to tx anaphalaxis;

D

The ICU nurse is caring for a patient who experienced trauma in a workplace accident. The patient is complaining of having trouble breathing with abdominal pain. An ABG reveals the following results: pH 7.28, PaCO2 50 mm Hg, HCO3- 23 mEq/L. The nurse should recognize the likelihood of what acid base disorder? A) Respiratory acidosis B) Metabolic alkalosis C) Respiratory alkalosis D) Mixed acid base disorder

C

The nurse is assessing the patient for the presence of a Chvostek's sign. What electrolyte imbalance would a positive Chvostek's sign indicate? A) Hypermagnesemia B) Hyponatremia C) Hypocalcemia D) Hyperkalemia

A

The nurse is caring for a patient in metabolic alkalosis. The patient has an NG tube to low intermittent suction for a diagnosis of bowel obstruction. What drug would the nurse expect to find on the medication orders? A) Cimetidine B) Maalox C) Potassium chloride elixir D) Furosemide

B

The nurse is caring for a patient with Addison's disease who is scheduled for discharge. When teaching the patient about hormone replacement therapy, the nurse should address what topic? A) The possibility of precipitous weight gain B) The need for lifelong steroid replacement C) The need to match the daily steroid dose to immediate symptoms D) The importance of monitoring liver function

C

The nurse is caring for a patient with a diagnosis of Addison's disease. What sign or symptom is most closely associated with this health problem? A) Truncal obesity B) Hypertension C) Muscle weakness D) Moon face

A

The nurse is providing care for a patient with chronic obstructive pulmonary disease. When describing the process of respiration the nurse explains how oxygen and carbon dioxide are exchanged between the pulmonary capillaries and the alveoli. The nurse is describing what process? A) Diffusion B) Osmosis C) Active transport D) Filtration

D

The nurse is providing care for an older adult patient whose current medication regimen includes levothyroxine (Synthroid). As a result, the nurse should be aware of the heightened risk of adverse effects when administering an IV dose of what medication? A) A fluoroquinalone antibiotic B) A loop diuretic C) A proton pump inhibitor (PPI) D) A benzodiazepine

C

The nurse is teaching a patient that the body needs iodine for the thyroid to function. What food would be the best source of iodine for the body? A) Eggs B) Shellfish C) Table salt D) Red meat

A

The nurse providing care for a patient with Cushing syndrome has identified the nursing diagnosis of risk for injury related to weakness. How should the nurse best reduce this risk? A) Establish falls prevention measures. B) Encourage bed rest whenever possible. C) Encourage the use of assistive devices. D) Provide constant supervision.

B

The nurse's assessment of a patient with thyroidectomy suggests tetany and a review of the most recent blood work corroborate this finding. The nurse should prepare to administer what intervention? A) Oral calcium chloride and vitamin D B) IV calcium gluconate C) STAT levothyroxine D) Administration of parathyroid hormone (PTH)

antihistamines

These are contraindicated in: in 3rd trimester, asthma, children and older people; pts with conditions in which muscarinic effects may exacerbate their underlying chronic conditions: asthma, urinary retention, open-angle glaucoma, BPH, HTN

atopic dermatitis

These are contraindicated in: in 3rd trimester, asthma, children and older people; pts with conditions in which muscarinic effects may exacerbate their underlying chronic conditions: asthma, urinary retention, open-angle glaucoma, BPH, HTN which often appears as a red, itchy rash normally on the cheeks, arms and legs.

B

While assisting with the surgical removal of an adrenal tumor, the OR nurse is aware that the patient's vital signs may change upon manipulation of the tumor. What vital sign changes would the nurse expect to see? A) Hyperthermia and tachypnea B) Hypertension and heart rate changes C) Hypotension and hypothermia D) Hyperthermia and bradycardia

A

You are developing a care plan for a patient with Cushing syndrome. What nursing diagnosis would have the highest priority in this care plan? A) Risk for injury related to weakness B) Ineffective breathing pattern related to muscle weakness C) Risk for loneliness related to disturbed body image D) Autonomic dysreflexia related to neurologic changes

A, C, E

You are doing discharge teaching with a patient who has hypophosphatemia during his time in hospital. The patient has a diet ordered that is high in phosphate. What foods would you teach this patient to include in his diet? Select all that apply. A) Milk B) Beef C) Poultry D) Green vegetables E) Liver

B

You are the surgical nurse caring for a 65-year-old female patient who is postoperative day 1 following a thyroidectomy. During your shift assessment, the patient complains of tingling in her lips and fingers. She tells you that she has an intermittent spasm in her wrist and hand and she exhibits increased muscle tone. What electrolyte imbalance should you first suspect? A) Hypophosphatemia B) Hypocalcemia C) Hypermagnesemia D) Hyperkalemia

Respiratory acidosis rationale

is always due to inadequate excretion of CO2 with inadequate ventilation, resulting in elevated plasma CO2 concentrations and, consequently, increased levels of carbonic acid. Occurs in emergency situations, such as acute pulmonary edema, aspiration of a foreign object, atelectasis, pneumothorax, overdose of sedatives, sleep apnea, administration of oxygen to a patient with chronic hypercapnia (excessive CO2 in the blood), severe pneumonia, and acute respiratory distress syndrome. Can also occur in diseases that impair respiratory muscles, such as muscular dystrophy, myasthenia gravis, and Guillain-BarrÈ syndrome

C

You are making initial shift assessments on your patients. While assessing one patient's peripheral IV site, you note edema around the insertion site. How should you document this complication related to IV therapy? A) Air emboli B) Phlebitis C) Infiltration D) Fluid overload

B

A 47-year-old patient who has come to the physician's office for his annual physical is being assessed by the office nurse. The nurse who is performing routine health screening for this patient should be aware that one of the first physical signs of aging is what? A) Having more frequent aches and pains B) Failing eyesight, especially close vision C) Increasing loss of muscle tone D) Accepting limitations while developing assets

C

A 52-year-old female patient is receiving care on the oncology unit for breast cancer that has metastasized to her lungs and liver. When addressing the patient's pain in her plan of nursing care, the nurse should consider what characteristic of cancer pain? A) Cancer pain is often related to the stress of the patient knowing she has cancer and requires relatively low doses of pain medications along with a high dose of anti-anxiety medications. B) Cancer pain is always chronic and challenging to treat, so distraction is often the best intervention. C) Cancer pain can be acute or chronic and it typically requires comparatively high doses of pain medications. D) Cancer pain is often misreported by patients because of confusion related to their disease process

D

A 60-year-old patient who has diabetes had a below-knee amputation 1 week ago. The patient asks ìwhy does it still feel like my leg is attached, and why does it still hurt?î The nurse explains neuropathic pain in terms that are accessible to the patient. The nurse should describe what pathophysiologic process? A) The proliferation of nociceptors during times of stress B) Age-related deterioration of the central nervous system C) Psychosocial dependence on pain medications D) The abnormal reorganization of the nervous system

D

A 73-year-old man comes into the emergency department (ED) by ambulance after slipping on a small carpet in his home. The patient fell on his hip with a resultant fracture. He is alert and oriented; his pupils are equal and reactive to light and accommodation. His heart rate is elevated, he is anxious and thirsty, a Foley catheter is placed, and 40 mL of urine is present. What is the nurse's most likely explanation for the low urine output? A) The man urinated prior to his arrival to the ED and will probably not need to have the Foley catheter kept in place. B) The man likely has a traumatic brain injury, lacks antidiuretic hormone (ADH), and needs vasopressin. C) The man is experiencing symptoms of heart failure and is releasing atrial natriuretic peptide that results in decreased urine output. D) The man is having a sympathetic reaction, which has stimulated the reninñangiotensinñaldosterone system that results in diminished urine output

D

A 74-year-old woman was diagnosed with rheumatoid arthritis 1 year ago, but has achieved adequate symptom control through the regular use of celecoxib (Celebrex), a COX-2 selective NSAID. The nurse should recognize that this drug, like other NSAIDs, influences what aspect of the pathophysiology of nociceptive pain? A) fibers B) Diverting noxious information from passing through the dorsal root ganglia and synapses in the dorsal horn of the spinal cord C) Blocking modulation by limiting the reuptake of serotonin and norepinephrine D) Inhibiting transduction by blocking the formation of prostaglandins in the periphery

C

A gerontologic nurse is aware of the demographic changes that are occurring in the United States, and this affects the way that the nurse plans and provides care. Which of the following phenomena is currently undergoing the most rapid and profound change? A) More families are having to provide care for their aging members. B) Adult children find themselves participating in chronic disease management. C) A growing number of people live to a very old age. D) Elderly people are having more accidents, increasing the costs of health care

D

A gerontologic nurse is basing the therapeutic programs at a long-term care facility on Miller's Functional Consequences Theory. To actualize this theory of aging, the nurse should prioritize what task? A) Attempting to control age-related physiological changes B) Lowering expectations for recovery from acute and chronic illnesses C) Helping older adults accept the inevitability of death D) Differentiating between age-related changes and modifiable risk factors

B

A gerontologic nurse is making an effort to address some of the misconceptions about older adults that exist among health care providers. The nurse has made the point that most people aged 75 years remains functionally independent. The nurse should attribute this trend to what factor? A) Early detection of disease and increased advocacy by older adults B) Application of health-promotion and disease-prevention activities C) Changes in the medical treatment of hypertension and hyperlipidemia D) Genetic changes that have resulted in increased resiliency to acute infection

D

A gerontologic nurse is overseeing the care that is provided in a large, long-term care facility. The nurse is educating staff about the significant threat posed by influenza in older, frail adults. What action should the nurse prioritize to reduce the incidence and prevalence of influenza in the facility? A) Teach staff how to administer prophylactic antiviral medications effectively. B) Ensure that residents receive a high-calorie, high-protein diet during the winter. C) Make arrangements for residents to limit social interaction during winter months. D) Ensure that residents receive influenza vaccinations in the fall of each year

A

A gerontologic nurse practitioner provides primary care for a large number of older adults who are living with various forms of cardiovascular disease. This nurse is well aware that heart disease is the leading cause of death in the aged. What is an age-related physiological change that contributes to this trend? A) Heart muscle and arteries lose their elasticity. B) Systolic blood pressure decreases. C) Resting heart rate decreases with age. D) Atrial-septal defects develop with age

C

A home health nurse makes a home visit to a 90-year-old patient who has cardiovascular disease. During the visit the nurse observes that the patient has begun exhibiting subtle and unprecedented signs of confusion and agitation. What should the home health nurse do? A) Increase the frequency of the patient's home care. B) Have a family member check in on the patient in the evening. C) Arrange for the patient to see his primary care physician. D) Refer the patient to an adult day program

B

A medical nurse is appraising the effectiveness of a patient's current pain control regimen. The nurse is aware that if an intervention is deemed ineffective, goals need to be reassessed and other measures need to be considered. What is the role of the nurse in obtaining additional pain relief for the patient? A) Primary caregiver B) Patient advocate C) Team leader D) Case manager

D

A newly graduated nurse is admitting a patient with a long history of emphysema. The new nurse's preceptor is going over the patient's past lab reports with the new nurse. The nurse takes note that the patient's PaCO2 has been between 56 and 64 mm Hg for several months. The preceptor asks the new nurse why they will be cautious administering oxygen. What is the new nurse's best response? A) The patient's calcium will rise dramatically due to pituitary stimulation. B) Oxygen will increase the patient's intracranial pressure and create confusion. C) Oxygen may cause the patient to hyperventilate and become acidotic. D) Using oxygen may result in the patient developing carbon dioxide narcosis and hypoxemia

C

A nurse has cited a research study that highlights the clinical effectiveness of using placebos in the management of postsurgical patients' pain. What principle should guide the nurse's use of placebos in pain management? A) Placebos require a higher level of informed consent than conventional care. B) Placebos are an acceptable, but unconventional, form of nonpharmacological pain management. C) Placebos are never recommended in the treatment of pain. D) Placebos require the active participation of the patient's family

B

A nurse in the neurologic ICU has orders to infuse a hypertonic solution into a patient with increased intracranial pressure. This solution will increase the number of dissolved particles in the patient's blood, creating pressure for fluids in the tissues to shift into the capillaries and increase the blood volume. This process is best described as which of the following? A) Hydrostatic pressure B) Osmosis and osmolality C) Diffusion D) Active transport

A

A nurse is caring for an 86-year-old female patient who has become increasingly frail and unsteady on her feet. During the assessment, the patient indicates that she has fallen three times in the month, though she has not yet suffered an injury. The nurse should take action in the knowledge that this patient is at a high risk for what health problem? A) A hip fracture B) A femoral fracture C) Pelvic dysplasia D) Tearing of a meniscus or bursa

C

A nurse is planning care for a nephrology patient with a new nursing graduate. The nurse states, ìA patient in renal failure partially loses the ability to regulate changes in pH.î What is the cause of this partial inability? A) The kidneys regulate and reabsorb carbonic acid to change and maintain pH. B) The kidneys buffer acids through electrolyte changes. C) The kidneys regenerate and reabsorb bicarbonate to maintain a stable pH. D) The kidneys combine carbonic acid and bicarbonate to maintain a stable pH

D

A nurse is planning discharge teaching for an 80-year-old patient with mild short-term memory loss. The discharge teaching will include how to perform basic wound care for the venous ulcer on his lower leg. When planning the necessary health education for this patient, what should the nurse plan to do? A) Set long-term goals with the patient. B) Provide a list of useful Web sites to supplement learning. C) Keep visual cues to a minimum to enhance the patient's focus. D) Keep teaching periods short

B

A nurse on an oncology unit has arranged for an individual to lead meditation exercises for patients who are interested in this nonpharmacological method of pain control. The nurse should recognize the use of what category of nonpharmacological intervention? A) A body-based modality B) A mind-body method C) A biologically based therapy D) An energy therapy

A

A nurse will conduct an influenza vaccination campaign at an extended care facility. The nurse will be administering intramuscular (IM) doses of the vaccine. Of what age- related change should the nurse be aware when planning the appropriate administration of this drug? A) An older patient has less subcutaneous tissue and less muscle mass than a younger patient. B) An older patient has more subcutaneous tissue and less durable skin than a younger patient. C) An older patient has more superficial and tortuous nerve distribution than a younger patient. D) An older patient has a higher risk of bleeding after an IM injection than a younger patient

A

A patient is experiencing severe pain after suffering an electrical burn in a workplace accident. The nurse is applying knowledge of the pathophysiology of pain when planning this patient's nursing care. What is the physiologic process by which noxious stimuli, such as burns, activate nociceptors? A) Transduction B) Transmission C) Perception D) Modulation

A

A patient who is being treated for pneumonia starts complaining of sudden shortness of breath. An arterial blood gas (ABG) is drawn. The ABG has the following values: pH 7.21, PaCO2 64 mm Hg, HCO3 = 24 mm Hg. What does the ABG reflect? A) Respiratory acidosis B) Metabolic alkalosis C) Respiratory alkalosis D) Metabolic acidosis

B

A patient's intractable neuropathic pain is being treated on an inpatient basis using a multimodal approach to analgesia. After administering a recently increased dose of IV morphine to the patient, the nurse has returned to assess the patient and finds the patient unresponsive to verbal and physical stimulation with a respiratory rate of five breaths per minute. The nurse has called a code blue and should anticipate the administration of what drug? A) Acetylcysteine B) Naloxone C) Celecoxib D) Acetylsalicylic acid

C

An 83-year-old woman was diagnosed with Alzheimer's disease 2 years ago and the disease has progressed at an increasing pace in recent months. The patient has lost 16 pounds over the past 3 months, leading to a nursing diagnosis of Imbalanced Nutrition: Less than Body Requirements. What intervention should the nurse include in this patient's plan of care? A) Offer the patient rewards for finishing all the food on her tray. B) Offer the patient bland, low-salt foods to limit offensiveness. C) Offer the patient only one food item at a time to promote focused eating. D) Arrange for insertion of a gastrostomy tube and initiate enteral feeding

C

An 84-year-old patient has returned from the post-anesthetic care unit (PACU) following hip arthroplasty. The patient is oriented to name only. The patient's family is very upset because, before having surgery, the patient had no cognitive deficits. The patient is subsequently diagnosed with postoperative delirium. What should the nurse explain to the patient's family? A) This problem is self-limiting and there is nothing to worry about. B) Delirium involves a progressive decline in memory loss and overall cognitive function. C) Delirium of this type is treatable and her cognition will return to previous levels. D) This problem can be resolved by administering antidotes to the anesthetic that was used in surgery

D

An elderly patient has come in to the clinic for her twice-yearly physical. The patient tells the nurse that she is generally enjoying good health, but that she has been having occasional episodes of constipation over the past 6 months. What intervention should the nurse first suggest? A) Reduce the amount of stress she currently experiences. B) Increase carbohydrate intake and reduce protein intake. C) Take herbal laxatives, such as senna, each night at bedtime. D) Increase daily intake of water

A

An elderly patient, while being seen in an urgent care facility for a possible respiratory infection, asks the nurse if Medicare is going to cover the cost of the visit. What information can the nurse give the patient to help allay her concerns? A) Medicare has a copayment for many of the services it covers. This requires the patient to pay a part of the bill. B) Medicare pays for 100% of the cost for acute-care services, so the cost of the visit will be covered. C) Medicare will only pay the cost for acute-care services if the patient has a very low income. D) Medicare will not pay for the cost of acute-care services so the patient will be billed for the services provided

C

An unlicensed nursing assistant (NA) reports to the nurse that a postsurgical patient is complaining of pain that she rates as 8 on a 0-to-10 point scale. The NA tells the nurse that he thinks the patient is exaggerating and does not need pain medication. What is the nurse's best response? A) ìPain often comes and goes with postsurgical patients. Please ask her about pain again in about 30 minutes.î B) ìWe need to provide pain medications because it is the law, and we must always follow the law.î C) ìUnless there is strong evidence to the contrary, we should take the patient's report at face value.'î D) ìIt's not unusual for patients to misreport pain to get our attention when we are busy.î

A

Based on a patient's vague explanations for recurring injuries, the nurse suspects that a community-dwelling older adult may be the victim of abuse. What is the nurse's primary responsibility? A) Report the findings to adult protective services. B) Confront the suspected perpetrator. C) Gather evidence to corroborate the abuse. D) Work with the family to promote healthy conflict resolution

B

Gerontologic nursing is a specialty area of nursing that provides care for the elderly in our population. What goal of care should a gerontologic nurse prioritize when working with this population? A) Helping older adults determine how to reduce their use of external resources B) Helping older adults use their strengths to optimize independence C) Helping older adults promote social integration D) Helping older adults identify the weaknesses that most limit them

D

Nurses and members of other health disciplines at a state's public health division are planning programs for the next 5 years. The group has made the decision to focus on diseases that are experiencing the sharpest increases in their contributions to the overall death rate in the state. This team should plan health promotion and disease prevention activities to address what health problem? A) Stroke B) Cancer C) Respiratory infections D) Alzheimer's disease

B

Older people have many altered reactions to disease that are based on age-related physiological changes. When the nurse observes physical indicators of illness in the older population, that nurse must remember which of the following principles? A) Potential life-threatening problems in the older adult population are not as serious as they are in a middle-aged population. B) Indicators that are useful and reliable in younger populations cannot be relied on as indications of potential life-threatening problems in older adults. C) The same physiological processes that indicate serious health care problems in a younger population indicate mild disease states in the elderly. D) Middle-aged people do not react to disease states the same as a younger population does

C

One day after a patient is admitted to the medical unit, you note that the patient is oliguric. You notify the acute-care nurse practitioner who orders a fluid challenge of 200 mL of normal saline solution over 15 minutes. This intervention will achieve which of the following? A) Help distinguish hyponatremia from hypernatremia B) Help evaluate pituitary gland function C) Help distinguish reduced renal blood flow from decreased renal function D) Help provide an effective treatment for hypertension-induced oliguria

B

The community health nurse is performing a home visit to an 84-year-old woman recovering from hip surgery. The nurse notes that the woman seems uncharacteristically confused and has dry mucous membranes. When asked about her fluid intake, the patient states, ìI stop drinking water early in the day because it is just too difficult to get up during the night to go to the bathroom.î What would be the nurse's best response? A) ìI will need to have your medications adjusted so you will need to be readmitted to the hospital for a complete workup.î B) ìLimiting your fluids can create imbalances in your body that can result in confusion. Maybe we need to adjust the timing of your fluids.î C) ìIt is normal to be a little confused following surgery, and it is safe not to urinate at night.î D) ìIf you build up too much urine in your bladder, it can cause you to get confused, especially when your body is under stress

A

The home health nurse is developing a plan of care for a patient who will be managing his chronic pain at home. Using the nursing process, on which concepts should the nurse focus the patient teaching? A) Self-care and safety B) Autonomy and need C) Health promotion and exercise D) Dependence and health

D

The home health nurse is making an initial home visit to a 76-year-old widower. The patient takes multiple medications for the treatment of varied chronic health problems. The patient states that he has also begun taking some herbal remedies. What should the nurse be sure to include in the patient's teaching? A) Herbal remedies are consistent with holistic health care. B) Herbal remedies are often cheaper than prescribed medication. C) It is safest to avoid the use of herbal remedies. D) There is a need to inform his physician and pharmacist about the herbal remedies

D

The mother of a cancer patient comes to the nurse concerned with her daughter's safety. She states that her daughter's morphine dose that she needs to control her pain is getting ìhigher and higher.î As a result, the mother is afraid that her daughter will overdose. The nurse educates the mother about what aspect of her pain management? A) The dose range is higher with cancer patients, and the medical team will be very careful to prevent addiction. B) Frequently, female patients and younger patients need higher doses of opioids to be comfortable. C) The increased risk of overdose is an inevitable risk of maintaining adequate pain control during cancer treatment. D) There is no absolute maximum opioid dose and her daughter is becoming more tolerant to the drug

A

The nurse caring for a 79-year-old man who has just returned to the medicalñsurgical unit following surgery for a total knee replacement received report from the PACU. Part of the report had been passed on from the preoperative assessment where it was noted that he has been agitated in the past following opioid administration. What principle should guide the nurse's management of the patient's pain? A) The elderly may require lower doses of medication and are easily confused with new medications. B) The elderly may have altered absorption and metabolism, which prohibits the use of opioids. C) The elderly may be confused following surgery, which is an age-related phenomenon unrelated to the medication. D) The elderly may require a higher initial dose of pain medication followed by a tapered dose

D

The nurse caring for a 91-year-old patient with osteoarthritis is reviewing the patient's chart. This patient is on a variety of medications prescribed by different care providers in the community. In light of the QSEN competency of safety, what is the nurse most concerned about with this patient? A) Depression B) Chronic illness C) Inadequate pain control D) Drug interactions

D

The nurse is accepting care of an adult patient who has been experiencing severe and intractable pain. When reviewing the patient's medication administration record, the nurse notes the presence of gabapentin (Neurontin). The nurse is justified in suspecting what phenomenon in the etiology of the patient's pain? A) Neuroplasticity B) Misperception C) Psychosomatic processes D) Neuropathy

B

The nurse is assessing a patient's pain while the patient awaits a cholecystectomy. The patient is tearful, hesitant to move, and grimacing. When asked, the patient rates his pain as a 2 at this time using a 0-to-10 pain scale. How should the nurse best respond to this assessment finding? A) Remind the patient that he is indeed experiencing pain. B) Reinforce teaching about the pain scale number system. C) Reassess the patient's pain in 30 minutes. D) Administer an analgesic and then reassess

C

The nurse is caring for a 51-year-old female patient whose medical history includes chronic fatigue and poorly controlled back pain. These medical diagnoses should alert the nurse to the possibility of what consequent health problem? A) Anxiety B) Skin breakdown C) Depression D) Hallucinations

A

The nurse is caring for a 65-year-old patient who has previously been diagnosed with hypertension. Which of the following blood pressure readings represents the threshold between high-normal blood pressure and hypertension? A) 140/90 mm Hg B) 145/95 mm Hg C) 150/100 mm Hg D) 160/100 mm Hg

D

The nurse is caring for a male patient whose diagnosis of bone cancer is causing severe and increasing pain. Before introducing nonpharmacological pain control interventions into the patient's plan of care, the nurse should teach the patient which of the following? A) Nonpharmacological interventions must be provided by individuals other than members of the healthcare team. B) These interventions will not directly reduce pain, but will refocus him on positive stimuli. C) These interventions carry similar risks of adverse effects as analgesics. D) Reducing his use of analgesics is not the purpose of these interventions

A

The nurse is caring for a patient with metastatic bone cancer. The patient asks the nurse why he has had to keep getting larger doses of his pain medication, although they do not seem to affect him. What is the nurse's best response? A) ìOver time you become more tolerant of the drug.î B) ìYou may have become immune to the effects of the drug.î C) ìYou may be developing a mild addiction to the drug.î D) ìYour body absorbs less of the drug due to the cancer.î

C

The nurse is preparing to insert a peripheral IV catheter into a patient who will require fluids and IV antibiotics. How should the nurse always start the process of insertion? A) Leave one hand ungloved to assess the site. B) Cleanse the skin with normal saline. C) Ask the patient about allergies to latex or iodine. D) Remove excessive hair from the selected site

A

The nurse is providing patient teaching to a patient with early stage Alzheimer's disease (AD) and her family. The patient has been prescribed donepezil hydrochloride (Aricept). What should the nurse explain to the patient and family about this drug? A) It slows the progression of AD. B) It cures AD in a small minority of patients. C) It removes the patient's insight that he or she has AD. D) It limits the physical effects of AD and other dementias

A

The nurse who is a member of the palliative care team is assessing a patient. The patient indicates that he has been saving his PRN analgesics until the pain is intense because his pain control has been inadequate. What teaching should the nurse do with this patient? A) Medication should be taken when pain levels are low so the pain is easier to reduce. B) Pain medication can be increased when the pain becomes intense. C) It is difficult to control chronic pain, so this is an inevitable part of the disease process. D) The patient will likely benefit more from distraction than pharmacologic interventions

B

The physician has ordered a peripheral IV to be inserted before the patient goes for computed tomography. What should the nurse do when selecting a site on the hand or arm for insertion of an IV catheter? A) Choose a hairless site if available. B) Consider potential effects on the patient's mobility when selecting a site. C) Have the patient briefly hold his arm over his head before insertion. D) Leave the tourniquet on for at least 3 minutes

C

The presence of a gerontologic advanced practice nurse in a long-term care facility has proved beneficial to both the patients and the larger community in which they live. Nurses in this advanced practice role have been shown to cause what outcome? A) Greater interaction between younger adults and older adults occurs. B) The elderly recover more quickly from acute illnesses. C) Less deterioration takes place in the overall health of patients. D) The elderly are happier in long-term care facilities than at home

B

You are admitting a patient to your rehabilitation unit who has a diagnosis of persistent, severe pain. According to the patient's history, the patient's pain has not responded to conventional approaches to pain management. What treatment would you expect might be tried with this patient? A) Intravenous analgesia B) long term intrathecal or epidural catheter C) oral analgesia D) intramuscular anesthesia

C

You are assessing an 86-year-old postoperative patient who has an unexpressive, stoic demeanor. When you enter the room, the patient is curled into the fetal position and your assessment reveals that his vital signs are elevated and he is diaphoretic. You ask the patient what his pain level is on a 0-to-10 scale that you explained to the patient prior to surgery. The patient indicates a pain level of ìthree or so.î You review your pain-management orders and find that all medications are ordered PRN. How would you treat this patient's pain? A) Treat the patient on the basis of objective signs of pain and reassess him frequently. B) Call the physician for new orders because it is apparent that the pain medicine is not working. C) Believe what the patient says, reinforce education, and reassess often. D) Ask the family what they think and treat the patient accordingly

B, C, D, E

You are caring for a 20-year-old patient with a diagnosis of cerebral palsy who has been admitted for the relief of painful contractures in his lower extremities. When creating a nursing care plan for this patient, what variables should the nurse consider? Select all that apply. A) Patient's gender B) Patient's comorbid conditions C) Type of procedure be performed D) Changes in neurologic function due to the procedure E) Prior effectiveness in relieving the pain

C

You are caring for a 65-year-old male patient admitted to your medical unit 72 hours ago with pyloric stenosis. A nasogastric tube placed upon admission has been on low intermittent suction ever since. Upon review of the morning's blood work, you notice that the patient's potassium is below reference range. You should recognize that the patient may be at risk for what imbalance? A) Hypercalcemia B) Metabolic acidosis C) Metabolic alkalosis D) Respiratory acidosis

A, B, D

You are caring for a patient admitted to the medical-surgical unit after falling from a horse. The patient states ìI hurt so bad. I suffer from chronic pain anyway, and now it is so much worse.î When planning the patient's care, what variables should you consider? Select all that apply. A) How the presence of pain affects patients and families B) Resources that can assist the patient with pain management C) The influence of the patient's cognition on her pain D) The advantages and disadvantages of available pain-relief strategies E) The difference between acute and intermittent pain

A

You are caring for a patient admitted with a diagnosis of acute kidney injury. When you review your patient's most recent laboratory reports, you note that the patient's magnesium levels are high. You should prioritize assessment for which of the following health problems? A) Diminished deep tendon reflexes B) Tachycardia C) Cool, clammy skin D) Acute flank pain

D

You are caring for a patient who has a diagnosis of syndrome of inappropriate antidiuretic hormone secretion (SIADH). Your patient's plan of care includes assessment of specific gravity every 4 hours. The results of this test will allow the nurse to assess what aspect of the patient's health? A) Nutritional status B) Potassium balance C) Calcium balance D) Fluid volume status

B

You are caring for a patient with late-stage Alzheimer's disease. The patient's wife tells you that the patient has now become completely dependent and that she feels guilty if she takes any time for herself. What outcomes would be appropriate for the nurse to develop to assist the patient's wife? A) The caregiver learns to explain to the patient why she needs time for herself. B) The caregiver distinguishes essential obligations from those that can be controlled or limited. C) The caregiver leaves the patient at home alone for short periods of time to encourage independence. D) The caregiver prioritizes her own health over that of the patient

A

You are caring for a patient with sickle cell disease in her home. Over the years, there has been joint damage, and the patient is in chronic pain. The patient has developed a tolerance to her usual pain medication. When does the tolerance to pain medication become the most significant problem? A) When it results in inadequate relief from pain B) When dealing with withdrawal symptoms resulting from the tolerance C) When having to report the patient's addiction to her physician D) When the family becomes concerned about increasing dosage

D

You are caring for an 82-year-old man who was recently admitted to the geriatric medical unit in which you work. Since admission, he has spoken frequently of becoming a burden to his children and ìstaying afloatî financially. When planning this patient's care, you should recognize his heightened risk of what nursing diagnosis? A) Disturbed thought processes B) Impaired social interaction C) Decisional conflict D) Anxiety

D

You are creating a nursing care plan for a patient with a primary diagnosis of cellulitis and a secondary diagnosis of chronic pain. What common trait of patients who live with chronic pain should inform your care planning? A) They are typically more comfortable with underlying pain than patients without chronic pain. B) They often have a lower pain threshold than patients without chronic pain. C) They often have an increased tolerance of pain. D) They can experience acute pain in addition to chronic pain

A

You are frequently assessing an 84-year-old woman's pain after she suffered a humeral fracture in a fall. When applying the nursing process in pain management for a patient of this age, what principle should you best apply? A) Monitor for signs of drug toxicity due to a decrease in metabolism. B) Monitor for an increase in absorption of the drug due to age-related changes. C) Monitor for a paradoxical increase in pain with opioid administration. D) Administer analgesics every 4 to 6 hours as ordered to control pain

B

You are part of the health care team caring for an 87-year-old woman who has been admitted to your rehabilitation facility after falling and fracturing her left hip. The patient appears to be failing to regain functional ability and may have to be readmitted to an acute-care facility. When planning this patient's care, what do you know about the negative effects of the stress associated with pain? A) Stress is less pronounced in older adults because they generally have more sophisticated coping skills than younger adults B) It is particularly harmful in the elderly who have been injured or who are ill. C) It affects only those patients who are already debilitated prior to experiencing pain. D) It has no inherent negative effects; it just alerts the person/health care team of an underlying disease process

C

You are performing an admission assessment on an older adult patient newly admitted for end-stage liver disease. What principle should guide your assessment of the patient's skin turgor? A) Overhydration is common among healthy older adults. B) Dehydration causes the skin to appear spongy. C) Inelastic skin turgor is a normal part of aging. D) Skin turgor cannot be assessed in patients over 70

B

You are providing care for an 82-year-old man whose signs and symptoms of Parkinson disease have become more severe over the past several months. The man tells you that he can no longer do as many things for himself as he used to be able to do. What factor should you recognize as impacting your patient's life most significantly? A) Neurologic deficits B) Loss of independence C) Age-related changes D) Tremors and decreased mobility

D

You are the case manager for a 35-year-old man being seen at a primary care clinic for chronic low back pain. When you meet with the patient, he says that he is having problems at work; in the past year he has been absent from work about once every 2 weeks, is short-tempered with other workers, feels tired all the time, and is worried about losing his job. You are developing this patient's plan of care. On what should the goals for the plan of care focus? A) Increase the patient's pain tolerance in order to achieve psychosocial benefits. B) Decrease the patient's need to work and increase his sleep to 8 hours per night. C) Evaluate other work options to decrease the risk of depression and ineffective coping. D) Decrease the time lost from work to increase the quality of interpersonal relationships and decrease anxiety

B

You are the emergency department (ED) nurse caring for an adult patient who was in a motor vehicle accident. Radiography reveals an ulnar fracture. What type of pain are you addressing when you provide care for this patient? A) Chronic B) Acute C) Intermittent D) Osteopenic

C

You are the home health nurse caring for a homebound client who is terminally ill. You are delivering a patient-controlled analgesia (PCA) pump to the patient at your visit today. The family members will be taking care of the patient. What would your priority nursing interventions be for this visit? A) Teach the family the theory of pain management and the use of alternative therapies. B) Provide psychosocial family support during this emotional experience. C) Provide patient and family teaching regarding the operation of the pump, monitoring the IV site, and knowing the side effects of the medication. D) Provide family teaching regarding use of morphine, recognizing morphine overdose, and offering spiritual guidance

C

You are the nurse caring for a 91-year-old patient admitted to the hospital for a fall. The patient complains of urge incontinence and tells you he most often falls when he tries to get to the bathroom in his home. You identify the nursing diagnosis of risk for falls related to impaired mobility and urinary incontinence. The older adult's risk for falls is considered to be which of the following? A) The result of impaired cognitive functioning B) The accumulation of environmental hazards C) A geriatric syndrome D) An age-related health deficit

A

You are the nurse caring for a postsurgical patient who is Asian-American who speaks very little English. How should you most accurately assess this patient's pain? A) Use a chart with English on one side of the page and the patient's native language on the other so he can rate his pain. B) Ask the patient to write down a number according to the 0-to-10 point pain scale. C) Use the Visual Analog Scale (VAS). D) Use the services of a translator each time you assess the patient so you can document the patient's pain rating

A

You are the nurse caring for an 85-year-old patient who has been hospitalized for a fractured radius. The patient's daughter has accompanied the patient to the hospital and asks you what her father can do for his very dry skin, which has become susceptible to cracking and shearing. What would be your best response? A) ìHe should likely take showers rather than baths, if possible.î B) ìMake sure that he applies sunscreen each morning.î C) ìDry skin is an age-related change that is largely inevitable.î D) ìTry to help your father increase his intake of dairy products.î

D

You are the nurse caring for an elderly patient who is being treated for community- acquired pneumonia. Since the time of admission, the patient has been disoriented and agitated to varying degrees. Appropriate referrals were made and the patient was subsequently diagnosed with dementia. What nursing diagnosis should the nurse prioritize when planning this patient's care? A) Social isolation related to dementia B) Hopelessness related to dementia C) Risk for infection related to dementia D) Acute confusion related to dementia

B

You are the nurse caring for an elderly patient with cardiovascular disease. The patient comes to the clinic with a suspected respiratory infection and is diagnosed with pneumonia. As the nurse, what do you know about the altered responses of older adults? A) Treatments for older adults need to be more holistic than treatments used in the younger population. B) The altered responses of older adults reinforce the need for the nurse to monitor all body systems to identify possible systemic complications. C) The altered responses of older adults define the nursing interactions with the patient. D) Older adults become hypersensitive to antibiotic treatments for infectious disease states.

C

You are the nurse caring for patients in the urology clinic. A new patient, 78 years old, presents with complaints of urinary incontinence. An anticholinergic is prescribed. Why might this type of medication be an inappropriate choice in the elderly population? A) Gastrointestinal hypermotility can be an adverse effect of this medication. B) Detrusor instability can be an adverse effect of this medication. C) Confusion can be an adverse effect of this medication. D) Increased symptoms of urge incontinence can be an adverse effect of this medication

D

You are the nurse caring for the 25-year-old victim of a motor vehicle accident with a fractured pelvis and a ruptured bladder. The nurse's aide (NA) tells you that she is concerned because the patient's resting heart rate is 110 beats per minute, her respirations are 24 breaths per minute, temperature is 99.1∞F axillary, and the blood pressure is 125/85 mm Hg. What other information is most important as you assess this patient's physiologic status? A) The patient's understanding of pain physiology B) The patient's serum glucose level C) The patient's white blood cell count D) The patient's rating of her

D

You are the nurse coming on shift in a rehabilitation unit. You receive information in report about a new patient who has fibromyalgia and has difficulty with her ADLs. The off-going nurse also reports that the patient is withdrawn, refusing visitors, and has been vacillating between tears and anger all afternoon. What do you know about chronic pain syndromes that could account for your new patient's behavior? A) Fibromyalgia is not a chronic pain syndrome, so further assessment is necessary. B) The patient is likely frustrated because she has to be in the hospital. C) The patient likely has an underlying psychiatric disorder. D) Chronic pain can cause intense emotional responses

A

You are the nurse evaluating a newly admitted patient's laboratory results, which include several values that are outside of reference ranges. Which of the following would cause the release of antidiuretic hormone (ADH)? A) Increased serum sodium B) Decreased serum potassium C) Decreased hemoglobin D) Increased platelets

B

You are the nurse in a pain clinic caring for an 88-year-old man who is suffering from long-term, intractable pain. At this point, the pain team feels that first-line pharmacological and nonpharmacological methods of pain relief have been ineffective. What recommendation should guide this patient's subsequent care? A) The patient may want to investigate new alternative pain management options that are outside the United States. B) The patient may benefit from referral to a neurologist or neurosurgeon to discuss pain-management options. C) The patient may want to increase his exercise and activities significantly to create distractions. D) The patient may want to relocate to long-term care in order to have his ADL needs met

B

You are the nurse planning an educational event for the nurses on a subacute medical unit on the topic of normal, age-related physiological changes. What phenomenon would you include in your teaching plan? A) A decrease in cognition, judgment, and memory B) A decrease in muscle mass and bone density C) The disappearance of sexual desire for both men and women D) An increase in sebaceous and sweat gland function in both men and women

D

You are working on a burns unit and one of your acutely ill patients is exhibiting signs and symptoms of third spacing. Based on this change in status, you should expect the patient to exhibit signs and symptoms of what imbalance? A) Metabolic alkalosis B) Hypermagnesemia C) Hypercalcemia D) Hypovolemia

C

You have just received report on a 27-year-old woman who is coming to your unit from the emergency department with a torn meniscus. You review her PRN medications and see that she has an NSAID (ibuprofen) ordered every 6 hours. If you wanted to implement preventive pain measures when the patient arrives to your unit, what would you do? A) Use a pain scale to assess the patient's pain, and let the patient know ibuprofen is available every 6 hours if she needs it. B) Do a complete assessment, and give pain medication based on the patient's report of pain. C) Check for allergies, use a pain scale to assess the patient's pain, and offer the ibuprofen every 6 hours until the patient is discharged. D) Provide medication as per patient request and offer relaxation techniques to promote comfort

C

Your patient is 12-hours post ORIF right ankle. The patient is asking for a breakthrough dose of analgesia. The pain-medication orders are written as a combination of an opioid analgesic and a nonsteroidal anti-inflammatory drug (NSAID) given together. What is the primary rationale for administering pain medication in this manner? A) To prevent respiratory depression from the opioid B) To eliminate the need for additional medication during the night C) To achieve better pain control than with one medication alone D) To eliminate the potentially adverse effects of the opioid


Related study sets

Chapter 4 - Downers, uppers downers, all arounders

View Set

Philosophy 1102 (Logic) - Ch. 12: Moral Arguments

View Set

Developmental Psychology Lifespan

View Set

Pharm. Chapter 13 CNS Stimulants and Related Drugs

View Set